On a multiple-choice test, each question has 4 possible answers. A student does not know the answers to three questions, so the student guesses. What is the probability that the student gets all three questions wrong? The probability that the student gets all three questions wrong is . What is the probability that the student gets all three questions right? The probability that the student gets all three questions right is .

Answers

Answer 1

Answer:

P(making 3 wrong guesses) = 27/64

P(making 3 right guesses) = 1/64

Step-by-step explanation:

Number of possible answers per question = 4

Number of right answer per question = 1

Number of wrong answers per question = (4 - 1) = 3

Probability = (Number of required outcomes / Number of Total possible outcomes)

A.) probability of guess 3 questions wrong :

Probability of making a wrong guess = (number of wrong options / total number of options

P(making a wrong guess) = 3/4

Therefore,

P(making 3 wrong guesses) = 3/4 × 3/4 × 3/4 = (27/64)

B.) probability of making 3 right guesses:

P(making a right guess) = (number of right options per question / total number of options per question)

P(making a right guess) = 1/4

P(making 3 right guesses) =(1/4 × 1/4 × 1/4) = 1/64


Related Questions

The line plot shows the result of a survey asking students how many hours they spent reading last week. How many students spent 5 or more hours reading?

Answers

Answer:

15 total students

Step-by-step explanation:

Based on the line plot picture that is attached below it can be calculated that a total of 15 students spent 5 or more hours last week reading. This can be easily calculated by adding all the x's on the marks of 5 hours or more. With both the boys and girl students combined the tally was the following

5 hours : 4 students

6 hours : 1 student

7 hours : 3 students

8 hours : 3 students

9 hours : 0 students

10 hours: 2 students

11 hours : 2 students

12 hours : 0 students

adding up to 15 total students

HELPPPPPP ITS ABOUT EQUATIONSSSS HELPPPPP Explanation needed HELPPPPP PLEASEE THIS IS THE LAST QUESTION LEFTTTTTTT

Answers

Answer:

C. the average total cost for the first month of a gym membership

Step-by-step explanation:

x=1 is for month 1,

the value of y includes 1-month fee and one off payment, so this is the average total for the first month of membership

Answer:

It is the average total cost for the first month of a gym membership

Step-by-step explanation:

y = 34.99x+49

The 49 is the cost to join the gym and the 34.99 is the monthly cost

Let x =1 which is the cost after one month

It includes the cost to join and the 1st month membership

It is the average total cost for the first month of a gym membership

please solve y = 3x - 1​

Answers

Step-by-step explanation:

i think question is not complete.

The circumference of the earth is given. 


Circumference of earth: 24,901 miles


What is the diameter of earth? Round your answer to the nearest tenth. Use 3.14 for π.


Answers

Answer:

7930.3 miles = d

Step-by-step explanation:

The circumference equals

C = pi *d

24901 = 3.14 d

Divide each side by 3.14

24901 / 3.14 = d

7930.254777 = d

Rounding to the nearest tenth

7930.3 =d

What is the slope of the graph? slope = -1/3 slope = -3 slope = 3 slope = 1/3

Answers

Answer:

The slope is -3

Step-by-step explanation:

This is so because the line in pointing down, leading to the fact that the slope is negative, and rise over run is 3/1......

Therefore, the slope of the line is -3

To find the slope of the line, first start with a point on the graph.

Let's use the point (0,3) which we call point A.

The other point we will call point B.

Now remember that slope can be found using the ratio rise/run

between any two points that are on that line.

To get from point A to point B along this line, we must

first go down 3 units so we say that our rise is -3.

From there we move 1 unit to the right so our run is 1.

So our slope or rise/run is -3/1 which reduces to -3.

8. Where will the hour hand of a clock stop if it starts:
a.
from 7 and turns through 1 right angle?
b. from 11 and turns through 3 right angles

can you plz say me the answer​

Answers

Answer:

a. 11

b. 9

Step-by-step explanation:

thats the answer

Solve for m.

3>m+8/5
plz helppppp

Answers

Answer:

m < 1.4

Step-by-step explanation:

Make [tex]\frac{8}{5}[/tex] a decimal.

1.6

3 > m + 1.6

m < 1.4

pleass more math help

Answers

Can’t answer this question

Please answer correctly !!!! Will mark brainliest !!!!!!!!!!!!

Answers

Answer:

[tex]x^2+10x+24[/tex]

Step-by-step explanation:

So just multiply the sides:

[tex]x^2+6x+4x+24[/tex]

Which is:

[tex]x^2+10x+24[/tex]

Please help me!!!!!!

Answers

Answer:

C

Step-by-step explanation:

The addition of the x^5 term makes it non proportional

Answer:

C.

Step-by-step explanation:

A. y = (6x + 3) - 3= 6x

B. y = - 15x

D. = - 1/3 x

All of these are proportional because they have general formula y = kx.

A man divides his 360 cattle between his son in the ratio 7: 6: 5. Find the smallest share​

Answers

360/18=20

20*7=140

20*6=120

20*5=100

Answer=140:120:100

Answer: smallest share is 100

Step-by-step explanation:

7:6:5

Add the numbers so 7+6+5=18

Divide 360 by 18 which equals 20

Multiply each ratio by 20 so...

7×20:6×20:5×20

140:120:100

Please help! Correct answer only, please! I need to finish this assgnment this week! Matrix P has a dimensions of 3 X 4 and Matrix Q has the dimensions 4 X 5. Determine the dimensions of the matrix PQ if it is possible. Explain why if it is not. (refer to video part 4) Group of answer choices A. Matrix PQ would have the dimensions 3 X 4 B. Matrix PQ would have the dimensions 4 X 5 C. Matrix PQ would have the dimensions 3 X 5 D. These matrices can not be multiplied because their dimension don't align.

Answers

Answer: C

Step-by-step explanation:

Your answer is correct! When you multiply 2 matrices, the inner dimensions have to be equal. If they are equal, the resulting dimensions would be the outer dimentions.

(3×4)(4×5)

Since the inner (bolded) dimensions are the same, in this case 4×4, we know these matrices can be multiplied.

Since these matrices can be multiplied, the product will be 3×5, the underlined outer dimensions.

Which angle in ADEF has the largest measure?

Answers

Answer:

F is the largest angle

Step-by-step explanation:

The largest angle is opposite the largest side.  The smallest angle is opposite the smallest side.

The largest side is 4 so the largest angle is F

What is the shape of the cross-section formed when a cylinder intersects a
plane as shown in the drawing?

Answers

Answer:

circle,

Step-by-step explanation:

disc in the middle vertically in this case is equal to the bases

The intersection is called an Oval. Hence the correct option is A. An oval in mathematics is a shape just like a circle but with an elongated outline like the shape of an egg.

What is a Cross-section?

A cross-section is a surface, an area that is created or exposed by executing a straight cut across or through a shape.

Cross-sections in technical drawings are used for depicting the internal view of an object that is three-dimensional.

Learn more about Cross-Sections are:

https://brainly.com/question/10511133

k (t) = 10t - 19
k(-7) =

Answers

Answer:

k(-7) = - 89

Step-by-step explanation:

k (t) = 10t - 19

Let t = -7

k(-7) =10 * -7 - 19

        =-70-19

        -89

Answer:

[tex]k (-7) = -89[/tex]

Step-by-step explanation:

[tex]k (t) = 10t - 19[/tex]

[tex]k (-7) = 10(-7) - 19[/tex]

[tex]k (-7) = -70 - 19[/tex]

[tex]k (-7) = -89[/tex]


Before graduating this year, a senior homeroom was given a survey. Of those surveyed, 24% felt they learned better at home. Of this group, 80% said they plan on taking an online course in college. Of the students who felt they did not learn better at home, 40% said they plan on taking an online course in college

Part A
What is the probability a person who does not plan on taking an online course felt they learned better at home?

A : 2/21
B : 24/125
C : 38/125
D : 19/31
E :None of these

Part B
What is the probability a person who does plan on taking an online course felt they did not learn better at home?

A : 2/21
B : 24/125
C : 38/125
D : 19/31
E : None of these

Answers

Answer:

(A) The correct option is (A).

(B) The correct option is (E).

Step-by-step explanation:

The events can be defined as follows:

X =  students felt they learned better at home

Y = students plan on taking an online course in college

The information provided is:

P (X) = 0.24

P (Y|X) = 0.80

P (Y|X') = 0.40

[tex]P(Y'|X)=1-P(Y|X)\\=1-0.80\\=0.20[/tex]

[tex]P(Y'|X')=1-P(Y|X')\\=1-0.40\\=0.60[/tex]

The Bayes' theorem states that the conditional probability of an event E[tex]_{i}[/tex] given that another event A has already occurred is:

[tex]P(E_{i}|A)=\frac{P(A|E_{i})P(E_{i})}{\sum {P(A|E_{i})P(E_{i})}}[/tex]

(A)

Compute the probability a person who does not plan on taking an online course felt they learned better at home as follows:

Use the Bayes' theorem.

[tex]P(X|Y')=\frac{P(Y'|X)P(X)}{P(Y'|X)P(X)+P(Y'|X')P(X')}[/tex]

              [tex]=\frac{0.20\times 0.24}{(0.20\times 0.24)+(0.60\times 0.76)}\\\\=0.09524\\\\\approx 0.095[/tex]

Thus, the probability a person who does not plan on taking an online course felt they learned better at home is 0.095 or 2/21.

(B)

Compute the probability a person who does plan on taking an online course felt they did not learn better at home as follows:

[tex]P(X'|Y')=1-P(X|Y')\\=1-0.095\\=0.905[/tex]

Thus, the probability a person who does plan on taking an online course felt they did not learn better at home is 0.905.

(3x5−2x4−5)−(2x4+x2−10) Subtract the two polynomials

Answers

Answer:

3x^5-4x^4-x^2+5

Step-by-step explanation:

(3x^5−2x^4−5)−(2x^4+x^2−10)

Distribute the minus sign

(3x^5−2x^4−5)−2x^4-x^2+10

Combine like terms

3x^5-4x^4-x^2+5

Hello!

Answer:

[tex]\boxed{ \bf 3x^5~-~4x^4~-~x^2~+~5}[/tex]

__________________________________Explanation:

(3[tex]x^{5}[/tex] - 2[tex]x^{4}[/tex] - 5) - (2[tex]x^{4}[/tex] + x² - 10)

Drop the brackets:

3[tex]x^{5}[/tex] - 2[tex]x^{4}[/tex] - 5 - 2[tex]x^{4}[/tex] - x² + 10

Combine Like Terms:

3[tex]x^{5}[/tex] - 2[tex]x^{4}[/tex] - 2[tex]x^{4}[/tex] - x² - 5 + 10

3[tex]x^{5}[/tex] - 4[tex]x^{4}[/tex] - x² + 5

How many digits will be in the quotient?
39 4,641

Answers

Answer:

the answer is 119 so three digits

Step-by-step explanation:

Answer:

3 digits

Step-by-step explanation:

4641/39=119

PLS HELP

Simplify the function f(x) =
3x
(81)
4
Then determine the key aspects of the function.
The initial value is
The simplified base is
The domain is
The range is

Answers

Answer:

Step-by-step explanation:

This follows the form

[tex]y=a(b)^x[/tex]

where a is the initial value and b is the base with the exponent.  Using that information, we can see that the initial value in our function is 1/3. Simplifying the base will take some work. Let's first rewrite this is a radical:

[tex]81^{\frac{3x}{4} }=\sqrt[4]{81^{3x} }[/tex]

Now let's break up 81 into its factors. 81 is 9*9 which is 3*3*3*3.  Therefore,

[tex]81=3^4[/tex]

We will use that as a simplification:

[tex]\sqrt[4]{(3^4)^{3x}}[/tex]  which simplifies to

[tex]\sqrt[4]{3^{12x}}[/tex]

Rewriting that as an exponent looks like this:

[tex]3^{\frac{12x}{4}}[/tex] which simplifies to

[tex]3^{3x}[/tex]

That's the answer for the second part.  The whole exponential equation now is

[tex]f(x)=\frac{1}{3}(3)^{3x}[/tex]

The domain for an exponential function is all real numbers and the range is

y > 0

Answer:

1/3

27

all real numbers

y > 0

ON EDGE

Step-by-step explanation:

You are going to use an incline plane to lift a heavy object to the top of a shelving unit with a height of 5 ft. The base of the incline plane is 15 ft from the shelving unit. What is the length of the incline​ plane?

Answers

Answer:

15.8 ft

Step-by-step explanation:

The inclined plane, the base of and the shelving unit form the shape of a right angled triangle.

The hypotenuse is the length of the inclined plane, h.

The base of the triangle is 15 ft.

The height of the triangle is 5 ft.

To find the hypotenuse, h, we have to use Pythagoras rule:

[tex]h^2 = a^2 + b^2[/tex]

where a = height of the triangle

b = base of the triangle

Therefore:

[tex]h^2 = 5^2 + 15^2\\\\h^2 = 25 + 225 = 250\\\\h = \sqrt{250}\\ \\h = 15.8 ft[/tex]

The inclined plane is 15.8 ft long.

Find the value of a . A.18 B.21 C.20 D.17

Answers

Answer:

a =18

Step-by-step explanation:

The two angles are vertical angles and vertical angles are equal

6a +11 = 2a+83

Subtract 2a from each side

6a-2a +11 = 2a-2a+83

4a +11 =83

Subtract 11 from each side

4a +11 -11 = 83-11

4a = 72

divide each side by 4

4a/4 = 72/4

a =18

En un programa de televisión se hacen 30 preguntas. Por cada respuesta correcta se suman 8 puntos, por cada respuesta errónea se restan 5 puntos y por aquellas preguntas que no se contesten no se suman ni se restan puntos. Si un participantes obtuvo 13 puntos, ¿Cuantas respuestas erróneas pudo tener?

Answers

Answer:

7 wrong answers.

Step-by-step explanation:

We know that when you answer a question well, you earn 8 points and a bad one loses 5 points, this means that when you answer 1 good and 1 bad, there is a total of 3 points (8-5).

This means that when answering 5 good and 5 bad, in total it would be 3 * 5 points, that is to say 15 points, that is, it goes through 2 points. To subtract 2 points, you would have to answer 1 good and 2 bad, (8 - 2 * 5), which turns out to be -2 points.

In total there would be 6 questions right and 7 questions wrong, like this:

8 * 6 + 5 * 7 = 13

13 points, in 13 questions (6 correct + 7 incorrect)

The rest of the questions were not answered so that the score does not go up or down.

Which means you got 7 wrong answers.

(6) Work out
5 1
6 12

Answers

Answer:

=123

Step-by-step explanation:

John has two jobs and earns a total of $2,345 per month. What percent of his gross income does John receive from his second job where he makes $609.70 a month?

Answers

Answer:

10000000.9

Step-by-step explanation:

The lateral area of a right cone which has a base diameter of 4 units and a height of 10 units is:

Answers

Answer:

≈64.08

Step-by-step explanation:

Lateral Area=πr[tex]\sqrt{h^{2} +r^{2} }[/tex]

Radius is 1/2 of the diameter so r=2

Height=10

Substitute:

: [tex]3.141(2)\sqrt{10^{2}+2^{2} } \\6.282\sqrt{100+4} \\6.282\sqrt{104} \\ 64.066.282*10.2\\ 64.076\\\\[/tex]

64.076 rounded = 64.08

Brass contains copper and zinc in the ratio 2:1. How much ZINC is there in 750 g of brass?

Answers

Answer:

250

Step-by-step explanation:

copper: zinc : total

2              1      2+1 =3

We have 750 brass

750/3 = 250

Multiply each by 250

copper: zinc : total

2*250     250  3*250

500        250       750

Jasper decided to save $100 at the end of each month for a year and deposit it in a bank account that earns an annual interest rate of 0.3%, compounded monthly. Use the formula for an annuity, F, to determine how much money will be in the account at the end of the 6th month, rounding your answer to the nearest penny.
Note: Your interest rate must be converted to a decimal

Answers

Answer: 600.38

Step-by-step explanation:

just put in the answer you lazy

Can someone please help

Answers

Answer:

The numbers to input are 2, 2 , 7

Step-by-step explanation:

252 = 2² * 3² * 7

So the numbers to input are 2, 2 , 7

Identify the like terms in the expression.

Answers

Answer:D

The answer is D.

Step-by-step explanation:

Like terms are only related to the variable at the end, and the variable also has to have the same exponent or it is not a like term.

What the answer to this

Answers

Answer:

I think it might be a I don't know for sure though I just need some more points so I can ask a question myself sorry if this didn't help

Other Questions
Diego has purchased a $159,000 home with a 30-year mortgage at 5.05%. Hecan make a monthly payment of $1300. If he were to make this payment eachmonth, how long will it take him to pay off his mortgage? Lauren drove The 75 miles to the coast at 75 mph, but gets stuck in traffic and averaged 25 mph on the way back. How many total miles did Lauren Drive in this round trip LiteraryLyricsName That Literary Device!Name 3 literary devices thatEd Sheeran uses from this lineof his song A-TeamBut lately her face seemsSlowly sinking, wastingCrumbling like pastries.Explanation You draw one card from a standard deck of playing cards. If you pick a heart, you win $10. If you pick a face card, which is not a heart, you win $8. If you pick any other card you lose $6. What is the expected value of the game? In the South, many enslaved African Americans supported the Union by _____.a. refusing to fight in the Confederate armyb. revolting against plantation ownersc. deliberately working faster to increase productiond. providing military information to Union armies if water were a nonpolar molecule, how would its properties be different? What are the vertex and x-intercepts of the graph of y= x2 - 4x - 21? Selectone answer for the vertex and one for the x-intercepts.A. Vertex: (-2, -9)B. x-intercepts: (3.0), (-7,0)C. Vertex: (2,-25)D. x-intercepts: (3.0), (7,0)E. x-intercepts: (-3,0), (7,0)F. Vertex: (2, 13) WILL GIVE BRAINLIEST ASAP NEEDED PLS ANSWER HELPYou need to create a fenced Off region of Land for cattle to graze. The grazing area must be a total of 500 square feet surrounded by a fence and in the shape of a regular polygon within this grazing area the length of the apothem must be 10 feet long.Part. 1 find the total perimeter of the grazing areaPart 2. if the cost of the fence is $7.95 per linear foot how much will it cost to place a fence around the entire grazing area?Part 3 - suppose this grazing area is 1/60 of an industrial grazing area for major industry farm. What is the total cost to build a fence around the entire land of the large scaled farm Use property of proportions show all work Represent each of the following variables in the appropriate letters.Next determine wether the variable has a fixed value or a changed value. a)The score Sarah got in a Mathematics test. b)The distance between the Zen house and the school. c)Temperature on the summit of Mount Everest in a day. Pacifica Industrial Products Corporation makes two products, Product H and Product L. Product H is expected to sell 40,000 units next year and Product L is expected to sell 8,000 units. A unit of either product requires 0.4 direct labor-hours. The company's total manufacturing overhead for the year is expected to be $1,632,000. Required: 1-a. The company currently applies manufacturing overhead to products using direct labor-hours as the allocation base. If this method is followed, how much overhead cost per unit would be applied to each product Statement of Shareholders' Equity You may use the attached spreadsheet to help you complete this activity, but you are not required to do so. You will find the spreadsheet by clicking on the green Excel icon in the upper left hand corner of the activity. On January 1, 2019, Powder Company provided the following shareholders' equity section of its balance sheet: Contributed Capital: Preferred stock, $100 par $ 92,800 Common stock, $5 par 37,400 Additional paid-in capital on preferred stock 21,500 Additional paid-in capital on common stock 58,700 Total contributed capital $210,400 Retained earnings 185,700 Total Shareholders' Equity $396,100 During 2019, the following transactions and events occurred and were properly recorded: Powder issued 1,800 shares of common stock at $13 per share. Powder issued 340 shares of preferred stock at $130 per share. Powder earned net income of $38,950. Powder paid a $7 per share dividend on the preferred stock and a $1 per share dividend on the common stock outstanding at the end of 2019. Required: Prepare Powder's statement of shareholders' equity (include retained earnings) for 2019. POWDER COMPANY Statement of Shareholders' Equity For Year Ended December 31, 2019 Preferred Stock $100 par Common Stock $5 par Additional Paid-in Capital on Preferred Stock Additional Paid-in Capital on Common Stock Retained Earnings Total $ $ $ $ $ $ $ $ $ $ $ $ Rate of Return, Return on Investment, and ROI all mean the same thing which best describes these terms Given f(x) = 6x + 2, find f(x-3) One study revealed that a child under the age of 10 watches television 4.5 hours per day. A group of families from a certain comunity would like to beleive that their children watch television less than the national average. A random sample of 14 children from the community yielded a mean of 4.1 hours per day with standard deviation of 1.2. Test the appropriate hypothesis at the level of significance 0.01. Assume the viewing time is normally distributed and interpret your results. Ashley has a limited supply of paper. Which print layout option willminimize the amount of paper used and includes up to nine images perpage? 29. Most prostate cancer occurs in ( )A Central zone8. Peripheral moneC. Transitional zoneD. Periurethral glandE. None of the above The physician changes a prescription for 150 mL of amoxicillin 250 mg/5 mL to 250 mg chewable tablets. How many chewable tablets should be dispensed William has an A.A. in general studies, but he does not know what career he wants to pursue. He decides to get a job for a year before going back to school. He wants a job near home in an office. He enjoys collaborating with other employees. William places a lot of value on freedom of thought and action at work. He needs about $50,000 per year.William interviews at a company as an entry level sales person. He learns that it takes about 15 minutes to drive to the office. His job would be to work in a cubicle on a phone talking to potential clients from a script. The job pays about $50,000 per year with the possibility for performance-based bonuses. Which factor makes this job a poor fit for William?A. The company is in a bad location.B. He wants more freedom in how he makes sales.C. He is opposed to sales work.D. The job does not pay enough money. 2+2x=3+2x explain why the equation does not have a solution Addicting drugs change your brain by giving it a rush from the strong release ofserotoninhepatitisdopaminehistamine